ab+1(a+b)2+bc+1(b+c)2+ca+1(c+a)2≥3\large{\frac { ab+1 }{ { ( a+b ) }^{ 2 } } +\frac { bc+1 }{ ( b+c ) ^{ 2 } } +\frac { ca+1 }{ { ( c+a ) }^{ 2 } } \ge 3}(a+b)2ab+1+(b+c)2bc+1+(c+a)2ca+1≥3
Let a,ba,ba,b and ccc be positive real numbers such that a2+b2+c2+(a+b+c)2≤4a^2+b^2+c^2+(a+b+c)^2 \le 4a2+b2+c2+(a+b+c)2≤4. Prove the above inequality.
Note by Department 8 5 years, 5 months ago
Easy Math Editor
This discussion board is a place to discuss our Daily Challenges and the math and science related to those challenges. Explanations are more than just a solution — they should explain the steps and thinking strategies that you used to obtain the solution. Comments should further the discussion of math and science.
When posting on Brilliant:
*italics*
_italics_
**bold**
__bold__
- bulleted- list
1. numbered2. list
paragraph 1paragraph 2
paragraph 1
paragraph 2
[example link](https://brilliant.org)
> This is a quote
This is a quote
# I indented these lines # 4 spaces, and now they show # up as a code block. print "hello world"
\(
\)
\[
\]
2 \times 3
2^{34}
a_{i-1}
\frac{2}{3}
\sqrt{2}
\sum_{i=1}^3
\sin \theta
\boxed{123}
Here is my solution for the problem:
Since
(a+b)2+(b+c)2+(c+a)2=2(a2+b2+c2+ab+bc+ca)=a2+b2+c2+(a+b+c)2, \begin{aligned} (a+b)^2 + (b+c)^2 + (c+a)^2 &= 2(a^2 + b^2 + c^2 + ab + bc + ca) \\ &= a^2 + b^2 + c^2 + (a + b + c)^2, \end{aligned}(a+b)2+(b+c)2+(c+a)2=2(a2+b2+c2+ab+bc+ca)=a2+b2+c2+(a+b+c)2,
Let
α=b+cβ=c+aγ=a+b \begin{aligned} \alpha &= b + c \\ \beta &= c + a \\ \gamma &= a + b \end{aligned}αβγ=b+c=c+a=a+b
This implies
a=β+γ−α2b=α+γ−β2c=α+β−γ2\begin{aligned} a &= \frac{\beta + \gamma - \alpha}2 \\ b &= \frac{\alpha + \gamma - \beta}2 \\ c &= \frac{\alpha + \beta - \gamma}2 \end{aligned}abc=2β+γ−α=2α+γ−β=2α+β−γ
With this change of variables, the constraint becomes
α2+β2+γ2≤4,\alpha^2 + \beta^2 + \gamma^2 \le 4,α2+β2+γ2≤4,
while the left side of the inequality we need to prove is now
γ2−(α−β)2+44γ2+α2−(β−γ)2+44α2+β2−(γ−α)2+44β2≥γ2−(α−β)2+α2+β2+γ24γ2+α2−(β−γ)2+α2+β2+γ24α2+β2−(γ−α)2+α2+β2+γ24β2=2γ2+2αβ4γ2+2α2+2βγ4α2+2β2+2γα4β2=32+αβ2γ2+βγ2α2+γα2β2.\begin{aligned} & \frac{\gamma^2 - (\alpha - \beta)^2 + 4}{4\gamma^2} + \frac{\alpha^2 - (\beta - \gamma)^2 + 4}{4\alpha^2} + \frac{\beta^2 - (\gamma - \alpha)^2 + 4}{4\beta^2} \ge \\ & \frac{\gamma^2 - (\alpha - \beta)^2 + \alpha^2 + \beta^2 + \gamma^2}{4\gamma^2} + \frac{\alpha^2 - (\beta - \gamma)^2 + \alpha^2 + \beta^2 + \gamma^2}{4\alpha^2} + \frac{\beta^2 - (\gamma - \alpha)^2 + \alpha^2 + \beta^2 + \gamma^2}{4\beta^2} = \\ & \frac{2\gamma^2 + 2\alpha\beta}{4\gamma^2} + \frac{2\alpha^2 + 2\beta\gamma}{4\alpha^2} + \frac{2\beta^2 + 2\gamma\alpha}{4\beta^2} = \\ & \frac32 + \frac{\alpha\beta}{2\gamma^2} + \frac{\beta\gamma}{2\alpha^2} + \frac{\gamma\alpha}{2\beta^2}. \end{aligned}4γ2γ2−(α−β)2+4+4α2α2−(β−γ)2+4+4β2β2−(γ−α)2+4≥4γ2γ2−(α−β)2+α2+β2+γ2+4α2α2−(β−γ)2+α2+β2+γ2+4β2β2−(γ−α)2+α2+β2+γ2=4γ22γ2+2αβ+4α22α2+2βγ+4β22β2+2γα=23+2γ2αβ+2α2βγ+2β2γα.
Therefore it remains to prove that
αβ2γ2+βγ2α2+γα2β2≥32.\frac{\alpha\beta}{2\gamma^2} + \frac{\beta\gamma}{2\alpha^2} + \frac{\gamma\alpha}{2\beta^2} \ge \frac32.2γ2αβ+2α2βγ+2β2γα≥23..
Which is obviously true by AM-GM.
Log in to reply
Great solution!
Thank you:-)
This is 2011 USAMO #1 I believe
Yes it is.
Given constraint: a^2+b^2+c^2+ab+bc+ca\leq2 Now, frac{(ab+1)/(a+b)^2}\geq1/2+1/2{(a+b)(b+c)}/(a+b)^2 Cyclic summing over three variables a,b,c And using A.M-G.M we obtain required result
Problem Loading...
Note Loading...
Set Loading...
Easy Math Editor
This discussion board is a place to discuss our Daily Challenges and the math and science related to those challenges. Explanations are more than just a solution — they should explain the steps and thinking strategies that you used to obtain the solution. Comments should further the discussion of math and science.
When posting on Brilliant:
*italics*
or_italics_
**bold**
or__bold__
paragraph 1
paragraph 2
[example link](https://brilliant.org)
> This is a quote
\(
...\)
or\[
...\]
to ensure proper formatting.2 \times 3
2^{34}
a_{i-1}
\frac{2}{3}
\sqrt{2}
\sum_{i=1}^3
\sin \theta
\boxed{123}
Comments
Here is my solution for the problem:
Since
(a+b)2+(b+c)2+(c+a)2=2(a2+b2+c2+ab+bc+ca)=a2+b2+c2+(a+b+c)2,
Let
αβγ=b+c=c+a=a+b
This implies
abc=2β+γ−α=2α+γ−β=2α+β−γ
With this change of variables, the constraint becomes
α2+β2+γ2≤4,
while the left side of the inequality we need to prove is now
4γ2γ2−(α−β)2+4+4α2α2−(β−γ)2+4+4β2β2−(γ−α)2+4≥4γ2γ2−(α−β)2+α2+β2+γ2+4α2α2−(β−γ)2+α2+β2+γ2+4β2β2−(γ−α)2+α2+β2+γ2=4γ22γ2+2αβ+4α22α2+2βγ+4β22β2+2γα=23+2γ2αβ+2α2βγ+2β2γα.
Therefore it remains to prove that
2γ2αβ+2α2βγ+2β2γα≥23..
Which is obviously true by AM-GM.
Log in to reply
Great solution!
Log in to reply
Thank you:-)
This is 2011 USAMO #1 I believe
Log in to reply
Yes it is.
Given constraint: a^2+b^2+c^2+ab+bc+ca\leq2 Now, frac{(ab+1)/(a+b)^2}\geq1/2+1/2{(a+b)(b+c)}/(a+b)^2 Cyclic summing over three variables a,b,c And using A.M-G.M we obtain required result